Verbal questions from any Manhattan Prep GMAT Computer Adaptive Test. Topic subject should be the first few words of your question.
sanketsao
Students
 
Posts: 2
Joined: Sat Jun 20, 2009 10:25 pm
 

Manhtaan 700-800 question

by sanketsao Tue Jul 27, 2010 3:31 pm

An economic recession can result from a lowering of employment rates triggered by a drop in investment, which causes people to cut consumer spending and starts a cycle of layoffs leading back to even lower employment rates.
a. a lowering of employment rates triggered by a drop in investment, which causes people to cut consumer spending and start a cycle of layoffs leading back to even lower employment rates.

b a lowering of employment rates triggered by dropping investment, which cause people to cut consumer spending and starts a cycle of layoffs leading back to even lower employment rates.

c. falling employment rates triggered by a drop in investment, which cause cutbacks in consumer spending and starting a cycle of layoffs that lead to even lower employment rates.

d. falling employment rates that are triggered by a drop in investment, causing people to cut consumer spending and starting a cycle of layoffs that lead back to even lower employment rates.

e. falling employment rates that are triggered by a drop in investment, that cause cutbacks in consumer spending and the start of a cycle of layoffs leading to even lower employment rates.

Code: Select All Code
OA: C

Explanation says:

(B) The use of "which" incorrectly suggests that "dropping investment" "causes people to cut consumer spending" when, in fact, the employment rates cause this phenomenon. Additionally, the phrase "causes people to cut consumer spending" is wordy and the use of "back" is redundant, as it is implied by the word "cycle".

(C) CORRECT. This choice makes clear, through the use of the plural verb "cause", that the employment rates are responsible for the cutbacks in spending.

However which should be Incorrect on both B&C, how is which correctly modifying in C and Incorrectly modifying in B?
tim
Course Students
 
Posts: 5665
Joined: Tue Sep 11, 2007 9:08 am
Location: Southwest Airlines, seat 21C
 

Re: Manhtaan 700-800 question

by tim Mon Aug 23, 2010 8:14 pm

You have simply copied the question incorrectly. Answer choice C does not contain the word "which"..
Tim Sanders
Manhattan GMAT Instructor

Follow this link for some important tips to get the most out of your forum experience:
https://www.manhattanprep.com/gmat/forums/a-few-tips-t31405.html